I've suggested (& published in 21 journal papers) a new theory called quantised inertia (or MiHsC) that assumes that inertia is caused by horizons damping quantum fields. It predicts galaxy rotation & lab thrusts without any dark stuff or adjustment. My University webpage is here, I've written a book called Physics from the Edge and I'm on twitter as @memcculloch. Most of my content is at patreon now: here

Tuesday 12 February 2013

A mechanism for inertia


The paper I submitted to EPL (Europhysics Letters) before Christmas on a more specific mechanism for inertia and MiHsC, was accepted yesterday. It is now available at the journal here for free, and also on the arxiv here). The abstract:

The property of inertia has never been fully explained. A model for inertia (MiHsC or quantised inertia) has been suggested that assumes that 1) inertia is due to Unruh radiation and 2) this radiation is subject to a Hubble-scale Casimir effect. This model has no adjustable parameters and predicts the cosmic acceleration, and galaxy rotation without dark matter, suggesting that Unruh radiation indeed causes inertia, but the exact mechanism by which it does this has not been specified. The mechanism suggested here is that when an object accelerates, for example to the right, a dynamical (Rindler) event horizon forms to its left, reducing the Unruh radiation on that side by a Rindler-scale Casimir effect whereas the radiation on the other side is only slightly reduced by a Hubble-scale Casimir effect. This produces an imbalance in the radiation pressure on the object, and a net force that always opposes acceleration, like inertia. A formula for inertia is derived, and an experimental test is suggested.

4 comments:

Alan said...
This comment has been removed by the author.
Anonymous said...

Since you are community-oriented enough to host a blog, I will help myself to the opportunity to link to my question about your paper here:

http://physics.stackexchange.com/questions/54313/does-unruh-radiation-replace-the-cosmic-horizon-radiation

I fumbled a little to begin with by conflating the cosmic microwave background with the cosmic event horizon. Currently, my question includes how we decide to construct the new cosmic horizon for the accelerating observer, and if the Hawking radiation shifts instantly or slowly from the old horizon to the new Horizon.

The central claim as to how inertia itself can be derived from the effects of the radiation remains beyond my level. For now, I hope that the Stack Exchange question can hash out a community perspective on these lower level confusions.

Mike McCulloch said...

This is what I replied on the Stack Exchange: "Thank you for your interesting question. The following is what I assumed in the paper. If you accelerate to the right, the Rindler horizon to your left is a boundary beyond which things are in principle unobservable for you. So, as soon as the nearer Rindler horizon forms, the far cosmic horizon behind it becomes unobservable and therefore (following the attitude of Mach) irrelevant. I assumed that the distance to the cosmic horizon on the right remains the same. I hope this answers your questions. It probably raises a lot of new ones!"

David Schroeder said...

Hi Mike: Wouldn't there be a problem with local inertia arising from Unruh radiation originating from light years away, as all radiation is restricted to the speed of light, and inertia is instantaneous (or at least seems to be)?